0 Daumen
328 Aufrufe

\( \sqrt{\sqrt[3]{\sqrt[4]{(\frac{1}{x^2})^{-12}}}} \) (es soll hoch -12 heißen)
ich habe erstmal versucht, die 2. Wurzel wegzubekommen, also mit 1/2 potenziert, dann die 3, also 1/3, dann die 4, also 1/4
kam dann auf ( ( ( (\( \frac{1}{x^2} \) )-12 )1/2 ) 1/3 )1/4
also (\( \frac{1}{x^2} \) ) -1/2
man muss aber irgendwie auf x kommen...habe ich da was falsch gemacht? oder fehlt vielleicht ein Schritt?

Avatar von

2 Antworten

0 Daumen
 
Beste Antwort

\( \left(\frac{1}{x^{2}}\right)^{-12}=\frac{1}{\left(\frac{1}{x^{2}}\right)^{12}}=x^{24} \)

\( \sqrt[4]{x^{24}}=x^{\frac{24}{4}}=x^{6} \)

\( \sqrt[3]{x^{6}}=x^{\frac{6}{3}}=x^{2} \)

\( \sqrt{x^{2}}=|x| \)

Avatar von 36 k
0 Daumen

Falls x positiv ist:

\(x^{-2*(-12)*\frac{1}{4*3*2}}=x^1=x\)

Avatar von 47 k

Ein anderes Problem?

Stell deine Frage

Willkommen bei der Mathelounge! Stell deine Frage einfach und kostenlos

x
Made by a lovely community